Answer (A) is correct . The first step is to calculate the contribution margin (CM) for a “composite” unit using budgeted mix percentages and budgeted margins: Product E: ?{[5,500 ÷ (5,500 + 4,500)] × $4.50} = $2.475 ? Product F: ?{[4,500 ÷ (5,500 + 4,500)] × $10.00} = $4.500 Composite Budget UCM $6.975 This process is repeated using actual mix percentages and budgeted margins: Product E: ?{[6,000 ÷ (6,000 + 6,000)] × $4.50} = $2.250 Product F: ?{[6,000 ÷ (6,000 + 6,000)] × $10.00} = $5.000 Composite Actual UCM $7.250 The difference between the two is multiplied by the number of units sold to arrive at the sales mix variance [(6,000 + 6,000) × ($7.250 actual – $6.975 budget) = (12,000 × $0.275) = $3,300 favorable].
Answer (B) is incorrect because Improperly using unweighted contribution margins results in $3,420 favorable. Answer (C) is incorrect because The amount of $17,250 is the sales volume variance. Answer (D) is incorrect because The sales volume variance in units multiplied by the actual price equals $18,150 favorable; it is not a mix variance.
|